You are on page 1of 9

Bartle - Introduction to Real Analysis - Chapter 9 Solutions

Section 9.1
Problem 9.1-1. Show that if a convergent series contains only a finite number of negative terms, then it is absolutely convergent.

xn , which converge and have a finite number of negative terms. Let (s0n )
P
Solution: Let (sn ) be P the partial sums of
be the partial sums of |xn |. It follows that for any  > 0, there is an M () ∈ N such that if m > n ≥ M (), then
|sm − sn | < . Let γ ∈ N be the greatest index of the negative terms of (xn ), and let M 0 () = sup{M (), γ}. Observe
that if j > M 0 (), then xj ≥ 0. Therefore, if m > n > M 0 (), then:
m m
X X
|s0m − s0n | = |xi | = xi = |sm − sn | < .


i=n+1 i=n+1
P P
Consequently, |xn | is convergent by the Cauchy Criterion for Series, and xn is absolutely convergent.

Problem 9.1-2. Show that if a series is conditionally convergent, then the series obtained from its positive terms is divergent,
and the series obtained from its negative terms is divergent.

Solution: Let (bn ) be a sequence where bn = (an + |an |)/2 and (cn ) is a sequence where cn = (an − |an |)/2, in each case
for all n ∈ N. It is clear thatP if an > 0, then bn = an and cn = 0; if an < 0, Pthen bn =P0 and cn = an ; and if an = 0, then
bn = cn P = 0. Accordingly, bn is the series from the positive terms of an and cn is the series from the negative
terms of an . Note that the zeros in (bn ) and (cn ) do not contribute to the partial sums and therefore do not affect their
convergence. P P P P
Suppose bn is convergent. It follows that |an | =
P2 bn − an , which is convergent because the two series
on the right-hand
P side are convergent. But by hypothesis a n is not absolutely convergent, so we have a contradiction.
Therefore, bn Pis divergent. P P P
Now suppose
P cn is convergent. We have |an | = an −2 cn , which is also convergent, leading to a contradiction.
Therefore, cn is divergent.

P∞
Problem 9.1-6. Find an explicit expression for the nth partial sum of n=2 ln(1 − 1/n2 ) to show that this series converges to
− ln 2. Is this convergence absolute?

xn where xn = ln(1 − 1/n2 ) for all n ∈ N where n ≥ 2. We can


P
Solution: Let (sn ) be the sequence of partial sums of
easily show by induction that sn = ln[(n + 1)/2n]. For n = 2, we have s2 = ln(3/4) = (2 + 1)/(2 · 2). If we assume the
inductive hypothesis holds for an n ∈ N, then:
     
1 n+1 n+2
sn+1 = xn+1 + sn = ln 1 − + ln = ln .
(n + 1)2 2n 2(n + 1)
The inductive hypothesis therefore holds for n ∈ N, n ≥ 2.
ln(1 − 1/n2 ) = lim ln[(n + 1)/2n] = ln(1/2). The series converges.
P
It follows that
Now observePthat xn < 0 2for all terms. Therefore |xn | = −xnPfor
all n ∈ N, n ≥ 2. If we let (pn ) be the sequence of
partial sums of ln(1 − 1/n ) , then pn = −sn . It follows that ln(1 − 1/n2 ) = lim pn = − lim sn = ln 2. Therefore,
ln(1 − 1/n2 ) converges absolutely.
P

P
Problem 9.1-9. If (an ) is a decreasing sequence of strictly positive numbers and if an is convergent, show that lim(nan ) = 0.

P
Solution: Since an is convergent, for a given /2 > 0 there is a H(/2) ∈ N such that if n/2 ≥ H(/2), then:
n

sn − sn/2 =
X 
ak < .
2
k=n/2+1
Pn
Since (an ) is strictly positive and decreasing, k=n/2+1 ak ≥ (n/2)an = 1/2 |nan |. Therefore, |nan | <  for n ≥
H(/2). Because  is arbitrary, lim(nan ) = 0.

Problem 9.1-11. If (an ) is a sequence and if lim(n2 an ) exists in R, show that


P
an is absolutely convergent.

Solution: Since lim(n2 an ) exists, (n2 an ) is bounded


P by some M > 0 such that n2 |an | ≤ M for all n ∈ N. As a P
result,
2 2
an ≤ M/n for all terms. ByP Example 3.7.6(c), M 1/n is convergent. Applying Theorem 3.7.7, it follows that |an |
is convergent. By definition, an is absolutely convergent.

Problem 9.1-13. (a) Does the series:


∞ √ √ 
X n+1− n

n=1
n
converge?
(b) Does the series:
∞ √ √ 
X n+1− n
n=1
n
converge?

Solution: Part (a) Let (an ) be a sequence where for all n ∈ N:

√ √ √ √
n+1− n n+1+ n 1 1 1 1 1
an = √ ·√ √ =√ √ √ >√ √ = √ > √ = .
n n+1+ n n( n + 1 + n) n(2 n + 1) 2
2 n +n 2 4n 2 4n
P
Clearly, an P
> 0 for all terms. We know that (1/4) 1/n is divergent as a harmonic series. Applying Theorem 3.7.7(a),
it follows that an is divergent.
Part (b) Let (bn ) be a sequence where for all n ∈ N:
√ √ √ √
n+1− n n+1+ n 1 1
bn = ·√ √ =√ 3 √ < √ = n−3/2
n n+1+ n 2
n +n +n n n n
P −3/2
Clearly, bn > 0 forPall terms. We know from Exercise 3.7.6(d) that n is convergent. Again applying Theorem
3.7.7(a), we see that bn is convergent.

P P P
Problem 9.1-14. If (an ) is a subsequence of (an ), then the series ank is called a subseries of an . Show that an is
absolutely convergent if and only if every subseries of its is convergent.

an is absolutley convergent with partial sums (s0n ). For any  > 0, there is an H() ∈ N such
P
Solution: First, suppose
that for m > n ≥ H():
m
X
|s0m − s0n | = |an | < .
k=n+1
P
Let (ank ) be any subsequence of (an ). Suppose (tn ) is the sequence ofPpartial sums of ank . For any p ∈ N, observe
n
that each ank summed in tp is also summed in s0np . It follows that tp ≤ i=1 p
|an | because all other terms in s0np not in
tp do not detract from the sum.
For some p, q ∈ N, if nq > np ≥ H(), then:

nq
q q
X X X
|an | = s0nq − s0np < .

|tq − tp | = ank ≤ |ank | ≤

k=p+1 k=p+1 k=np +1
P
By the Cauchy Criterion for series, ank is convergent.

Page 2
Conversely, suppose for every subsequence (an ), the associated subseries is convergent. Let (ank ) be a subsequence
including all terms of (an ) such that an ≥ 0, and let (aml ) be a subsequence including all terms of (an ) such that an < 0.
Observe that we can rewrite any term of (sn ) as:
X X
sn = ank + aml .
nk ≤n ml ≤n

Let /2 > 0 be given. There is a G(/2) ∈ N such that for j, k ∈ N where nj > nk ≥ G(/2):
j j
X X 
ani = |ani | < .

2


i=k+1 i=k+1

There is also a a J(/2) ∈ N such that for p, q ∈ N where mp > mq ≥ J(/2):



p p
X X 
am i
= |ami | < .

i=q+1

i=q+1 2

Now let K() = sup{G(/2), J(/2)}. Suppose v > u ≥ K(). We can find indices for the two subsequences that
collectively make up all terms of (an ) between indices v and u as follows:

k = inf{i ∈ N : ni ≥ u},
j = sup{i ∈ N : ni ≤ v},
q = inf{i ∈ N : mi ≥ u},
p = sup{i ∈ N : mi ≤ v}.
We then have:
m
X j
X p
X
|s0m − s0n | = |ai | = |ani | + |ami | < .
i=u+1 i=k+1 i=q+1
P
It follows from the Cauchy Criterion for series that an converges absolutely.

Problem 9.1-15. [See problem in text]

P P P∞ P∞
Solution: Suppose j i aij = j=1 i=1 aij converges P P to B.PObserve P that (ck ) is a rearrangement of (aij ). Since
aij ≥ 0 for all i, j ∈ N, it follows that aij = |aij |, so j i aij = j i |aij |. Therefore, the series converges absolutely.
P∞
Applying TheoremP9.1.5, wePinfer that k=1 ck converges to C, and C = B.

P Now suppose k ck = k=1 ck converges to C. Since ck = aij for some i, j P ∈ N,Pwe have ck = |ck |. Consequently,
k ck converges absolutely. Since (aij ) is a rearrangement of (ck ), it follows that j i aij converges to B, and B = C.

Section 9.2
Problem 9.2-1. Establish the convergence or the divergence of the series whose nth term is: (a) 1/[(n + 1)(n + 2)], (b)
n/[(n + 1)(n + 2)],(c) 2( 1/n), (d) n/2n .

Solution: Part (a) Let yn = 1/[(n + 1)(n + 2)] and xn = 1/n2 for all n ∈ N. We then have:

n2 + 3n + 2
 
xn 3 2
r = lim = lim
= lim 1 + + 2 = 1.
yn n2 n n
2
P P
Since 1/n is absolutely convergent, 1/[(n + 1)(n + 2)] is absolutely convergent by the Limit Comparison Test II.
Part (b) Let yn = n/[(n + 1)(n + 2)] and xn = 1/n for all n ∈ N. We then have:

n2 + 3n + 2
 
xn 3 2
r = lim = lim = lim 1 + + = 1.
yn n2 n n2

Page 3
P P
Since 1/n is divergent, it follows from the Limit Comparison Test II that n/[(n + 1)(n + 2)] is divergent, as well.
Part (c) Let xn = 2−1/n = (1/2)1/n for all nP ∈ N. From Example 3.1.11(c), lim(1/2)1/n = 1. Therefore, lim xn 6= 0,
from which it follows from nth Term Test that 2−1/n is divergent.
n
Part (d) Let xn = n/2 . We then have:
 
xn+1
r = lim = lim n + 1 = 1 lim 1 + 1 = 1 .
xn 2n 2 2n 2
n
P
Since xn 6= 0 for any term, by the Ratio Test n/2 is absolutely convergent.

Problem 9.2-2. Establish the convergence or the divergence of the series whose nth term is: (a) (n(n + 1))−1/2 , · · · , (c)
n!/nn , · · · .

Solution: Part(a) Let xn = (n(n + 1))−1/2 and yn = 1/n for all n ∈ N. We have:
r
xn n
r = lim = lim = 1.
yn n+1
1/n is divergent, by the Limit Comparison Test II, (n(n + 1))−1/2 is divergent.
P P
Since
Part (c) Let xb = n!/nn for all n ∈ N. Then:
 n
xn+1 n 1
lim
= lim = < 1,
xn n+1 e
n!/nn is absolutely convergent.
P
from Exercise 3.3.6. Applying the Ratio Test, we see that

Problem 9.2-3. Discuss the convergence or the divergence of the series whose nth term (for sufficiently large n): (a) (ln n)−p ;
(b) (ln n)−n ; (c) (ln n)− ln n ; (d) (ln n)− ln ln n ; (e) (n ln n)−1 ; · · ·

Solution: Part (a) Let xn = (ln n)−p for some p ∈ R for all n ∈ N. Let yn = 1/n. Applying L’Hopı̂tal’s Rule, we find:
xn n
lim = lim = 0.
n→∞ yn n→∞ (ln n)p
(ln n)p < n, from which it follows that 1/n < (ln n)−p for large n. From Theorem 3.7.7(b), since
P
Therefore, P 1/n is
divergent, (ln n)−p is divergent.
Part (b) Let xn = 1/(ln n)n . For n ≥ 3, we have:

1 1/n

1 1
0<

n
= ≤ ≈ 0.92.
(ln n) ln n ln 3
By the Root Test, (ln n)−n is absolutely convergent.
P
Part (c) Let xn = (ln n)− ln n for all n ∈ N. We can manipulate 1/xn as follows:

(ln n)ln n = (eln(ln n) )ln n = (eln n )ln(ln n) = nln(ln n) .


Comparing this to n2 , we see that nln(ln n) <Pn2 for ln(ln n) > 2, or
Pn > 1618. Therefore, 1/n2 > (ln n)− ln n for n > 1618.
2 − ln n
Applying Theorem 3.7.7, we see that since 1/n is divergent, (ln n) is divergent.
Part (d) We see that:
1 1
0< < ,
(ln n)ln(ln n) (ln n)ln n
1/(ln n)ln(ln n) is convergent.
P
for n ≥ 2. From the result in part (c) and Theorem 3.7.7,
Part (e) Let xn = 1/(n ln n) for n ∈ N. Note that 0 < xn /xn+1 = (n ln n)/((n + 1) ln(n + 1)) < 1, so (xn ) is
decreasing. We then have:
Z b Z ln b ln b  
1 1 ln b
lim dt = lim du = lim ln u = lim ln ,

b→∞ 1 t ln t b→∞ 0 u b→∞ 0 b→∞ 0
wherePwe used the substitution u = ln t to evaluate the integral. Since this improper integral does not exist, by the Integral
Test (n ln n)−1 is divergent.

Page 4
Problem 9.2-4. Discuss the convergence or the divergence of the series whose nth term: · · · (d) (ln n)− ln ln n , (e) n!en , (f)
2
n!e−n .

√ √ √
Solution: Part (d) Observe that (ln n)e− n
< ne− n
for all n ∈ N. Let u = n. We can evaluate the following limit
using L’Hopı̂tal’s Rule:

ne− n n3 u6 6!
lim 2
= lim √n = lim u = lim u = 0.
n→∞ 1/n n→∞ e n→∞ e n→∞ e
P −√n
Applying the√Limit Comparison Test II, we infer that ne is absolutely convergent. It follows from Theorem 3.7.7
that (ln n)e− n is convergent.
P
Part (e) Let xn = n!e−n for all n ∈ N. Then |xn+1 /xn | = (n + 1)/e > 1 for n ≥ 2. By the Ratio Test, n!e−n is
P
divergent.
2
Part (f) Let xn = n!e−n for all n ∈ N. Then |xn+1 /xn | = (n + 1)/e2n+1 . Applying L’Hopı̂tal’s Rule, we have:
n+1 1
lim = lim = 0.
n→∞ e2n+1 n→∞ 2e2n+1
2
n!e−n is absolutely convergent. Note that the answer in the back of the book is wrong!
P
By Corollary 9.2.1,

Problem 9.2-5. Show that the series 1/12 + 1/23 + 1/43 + · · · is convergent, but that both the Ratio and the Root Tests fail
to apply.

Solution: Let (an ) be a sequence where an = 1/n2 for odd n and an = P 1/n3 for even n. Observe that an ≤ 1/n2 for
2 2
all n ∈ N because
P an = 1/n for odd n and an < 1/n for even n. Since 1/n2 is convergent, it follows from Theorem
3.7.7 that an is convergent.
Now observe that if n is even, then |xn+1 /xn | = n2 /(n + 1)3 < 1; if n is odd, then |xn+1 /xn | = n3 /(n + 1)2 > 1. The
Ratio Test therefore does not apply. Moreover, because lim x2n+1 6= lim x2n , the limit does not exist for Corollary 9.2.5.
1/n 1/n 1/n 1/n
Also observe that |x2n | = 1/(2n)2/n and |x2n+1 | = 1/(2n+1)3/n . It follows that lim |x2n | = lim |x2n+1 | =
1/n
1. Therefore, lim |xn | = 1, so the Ratio Test is inconclusive.

(an + b)−p converges if p > 1 and diverges if p ≤ 1.


P
Problem 9.2-6. If a and b are positive numbers, then

Solution: Let xn = (an + b)−p for all n ∈ N where p ∈ R and a, b > 0. Consequently, xn > 0 for all terms.
If p < 0,Pthen xn = (an + b)α > 1 where α = −p. Therefore, it cannot be that lim xn = 0. Accordingly, from nth
Term Test, xn is divergent.
Now suppose p ≥ 0. Let yn = 1/np . We then have:
p p
(an + b)p
 
yn an + b b
r = lim = lim = lim = lim a + = ap > 0.
xn np n n
p
P P
P If pp > 1, then 1/n
P is convergent, so xn is convergent for the Limit Comparison Test II. If 0 ≤ p ≤ 1, then
1/n is divergent, so xn is divergent.

Problem 9.2-7. [See question in text]

Solution: Part (a) Let xn = n!/[3 · 5 · 7 · · · (2n + 1)] for all n ∈ N. Then:

xn+1 n+1 1 + 1/n
xn = 2n + 1 = 2 + 1/n .

P
Therefore, lim |xn+1 /xn | = 1/2. By the Ratio Test, xn is convergent.
Part (b) Let xn = (n!)2 /(2n)! for all n ∈ N. Then:

(n + 1)2 1 + 2/n + 1/n2



xn+1
= = .
xn (2n + 2)(2n + 1) 4 + 6/n + 3/n2

Page 5
P
Therefore, lim |xn+1 /xn | = 1/4. By the Ratio Test, xn is convergent.
Part (c) Let xn = [2 · 4 · · · (2n)(2n + 2)]/[3 · 5 · · · (2n + 1)] for all n ∈ N. We then have:

xn+1 2n + 2
xn = 2n + 3 .

A simple calculation shows that 1 − (2n + 2)/(2n + 3) = 1/(2n + 3) < 1/n. It follows that (2n + 2)/(2n + 3) > 1 − 1/n,
so:

xn+1 1
xn > 1 − n ,

P
for all n ∈ N. By Raabe’s Test, xn is divergent.
Part (d) Let xn = [2 · 4 · · · (2n)]/[5 · 7 · · · (2n + 3)] for all n ∈ N. We then have:

xn+1 2n + 2
xn = 2n + 5 .

A simple calculation shows that 1 − (2n + 2)/(2n + 5) = 3/(2n + 5) ≥ 6/(5n) for n ≥ 10. It follows that if n ≥ 10:

xn+1 6
xn ≤ 1 − 5n .

P
Applying Raabe’s Test, we see xn is absolutely convergent.

P
Problem 9.2-9. If r ∈ (0, 1) satisfies (2) in the Root Test 9.2.2, show that the partial sums sn of xn approximate its limit
s according to the estimate |s − sn | ≤ rn+1 /(1 − r) for n ≥ K.

1/(n+1)
Solution: By hypothesis, if n ≥ K for some K ∈ N, then |xn+1 | ≤ r. It follows that |xn+1 | ≤ rn+1 < 1 since
r ∈ (0, 1). Summing up terms from n + 1 to m for some m > n ≤ K, we have:
m
X rn+1 − rm+1
|xm | + |xm−1 | + · · · + |xn+1 | ≤ rk = .
1−r
k=n+1

Applying the Triangle Inequality:

rn+1 − rm+1
|sm − sn | = |xm + xm−1 + xn+1 | ≤ .
1−r
Taking the limit as m → ∞:
rn+1
|s − sn | ≤ ,
1−r
for n ≥ K.

Problem 9.2-15. For n ∈ N, let cn be defined by cn := 1/1 + 1/2 − 1/3 + 1/4 + 1/5 − 1/6 + · · · + 1/n − ln n. Show that
(cn ) is a decreasing sequence of positive numbers. Show that if:
1 1 1 1
bn := − + ··· − ,
1 2 3 2n
then the sequence (bn ) converges to ln 2. [Hint: bn = c2n − cn + ln 2,]

Solution: Observe that c1 = 1 − ln 1 = 1, c2 = 1 + 1/2 − ln 2 ≈ 0.807, and c2 − c1 ≈ −0.193.


We then have cn+1 − cn = 1/(n + 1) − ln(1 + 1/n). A simple calculation shows that (cn+1 − cn )0 = 1/[n(n + 1)2 ] > 0
for all n ∈ N. Therefore, (cn+1 − cn ) is strictly increasing from less than zero. Further, lim(cn+1 − cn ) = 0. By the
Monotone Convergence Theorem, 0 = sup{cn+1 − cn : n ∈ N}, so (cn+1 − cn ) is bounded above by 0. Therefore, (cn ) is
decreasing (in fact, strictly decreasing since (cn+1 − cn ) is strictly increasing).
We know that lim cn = γ ≈ 0.55721 (Euler’s constant). Since (cn ) is decreasing, γ = inf{cn : n ∈ N}. Therefore, (cn )
is bounded below by γ. Therefore, cn > 0 for all n ∈ N.

Page 6
We may express (bn ) as bn = c2n − cn − ln 2 for all terms. Observe that:
2k n n n n
X X 1 X 1 X 1 X 1
c2n − cn = 1/k − − ln 2 = + − − ln 2 =
k 2k − 1 2k k
k=1 k=1 k=1 k=1 k=1
n n   n  
X 1 X 1 1 X 1 1
= + − − ln 2 = − − ln 2
2k − 1 2k k 2k − 1 2k
k=1 k=1 k=1

= bn − ln 2.
It follows that lim bn = lim c2n − lim cn + ln 2 = ln 2 because lim c2n = lim cn .

Problem 9.2-18. Suppose that none of the numbers a, b, c is a negative integer or zero. Prove that the hypergeometric
series
ab a(a + 1)b(b + 1) a(a + 1)(a + 2)b(b + 1)(a + 2)
+ + + ···
1!c 2!c(c + 1) 3!c(c + 1)(c + 2)
is absolutely convergent for c > a + b and divergent for c < a + b.

Solution: Let (xn ) be a sequence of the terms of the hypergeometric series. For all n ∈ N:

xn+1 (a + n)(b + n)
xn = (n + 1)(c + n) .

We then have:
     
xn+1 (n + 1)(c + n) − (a + n)(b + n) n(c − a − b) + (c + n)
n 1−
=n =n
xn (n + 1)(c + n) (n + 1)(c + n)
 
1 c−a−b
= 1+ .
1 + 1/n 1 + c/n
Taking the limit, we have:  
xn+1
r = lim n 1 − = 1 + (c − a − b).
xn
P
If c > a + b, then c − a − b > 0, and r > 1. It follows from Raabe’s Test that xn is absolutely
Pconvergent.
On the other hand, if c < a + b, then c − a − b < 0,Pso r < 1. It follow from Raabe’s Test that xn is not absolutely
convergent. Since (xn ) is a strictly positive sequence, xn is divergent.

Section 9.3

Section 9.4
P P
Problem 9.4-2. If an is an absolutely convergent series, then the series an sin nx is absolutely and uniformly convergent.

P
Solution: Suppose an is absolutely convergent, where (sn ) is the sequence of partial sums of this series. For a given
 > 0, there is a K() ∈ N such that for m > n ≥ K():

|sm − sn | = |an+1 | + |an+2 | + · · · + |am | < .


By the Triangle Inequality, |an+1 + an+2 + · · · + am | < . P
Observe that |sin nx| ≤ 1 for all n ∈ N and all x ∈ R. Let (tn ) be the sequence of partial sums of an sin nx. If
m > n ≥ K():

|tm − tn | = |an+1 sin x(n + 1) + an+2 sin x(n + 2) + · · · + am sin xm|


≤ |an+1 sin x(n + 1)| + |an+2 sin x(n + 2)| + · · · + |am sin xm|

Page 7
≤ |an+1 | + |an+2 | + · · · + |am | < ,
for all x ∈ R. P
FromPthe Cauchy Criterion 9.4.5, an sin nx is uniformly convergent on R. In addition, by the Cauchy Criterion for
Series, an sin nx is absolutely convergent.

an xn is given by lim(|an /an+1 )| whenever this


P
Problem 9.4-5. Show that the radius of convergence R of the power series
limit exists. Give an example of a power series where this limit does not exist.

Solution: Assume an 6= 0 for all n ∈ N. Let  > 0 be given. Assuming that the series converges on some interval of N,
1/n 1/n
we have ρ > 0. Because ρ = lim sup |an | , by definition there is a K(/2) such that ρ + /2 ≥ |an | for n ≥ K(/2.
1/(n+1
For the same reason, ρ + /2 ≥ |an+1 | for n ≥ K(/2). Raising each to the n and n + 1 power, respectively, and
dividing, we get:

(ρ + /2)n+1

an+1
= ρ + /2 = .
(ρ + /2)n an
We then have for n ≥ K(/2):

an+1 an+1
 > /2 ≥ |ρ + /2| + |−ρ| ≥
+ |−ρ| ≥
− ρ .
an an
Accordingly, ρ = lim |an+1 /an |. It follows that R = 1/ρ = lim |an /an+1 |.

(1 + (−1)n )xn because some an /an+1 do not exist.


P
This limit does not exist for

an xn .
P
Problem 9.4-9. If 0 < p ≤ an ≤ q for all n ∈ N, find the radius of convergence of

1/n 1/n 1/n


Solution: Let ρ = lim sup |an | = lim um where um = sup{|an | : n ≥ m}. Since 0 < p1/n ≤ an and lim p1/n = 1,
1/n
it follows that um ≥ 1. In addition, since an ≤ q 1/n , it follows that um ≤ 1. By the Squeeze Theorem, ρ = lim um = 1.
Therefore, R = 1/ρ = 1.

an xn for |x| < R. If f (x) = f (−x) for all |x| < R, show that an = 0 for all odd n.
P
Problem 9.4-10. Let f (x) =

Solution: By hypothesis, if |x| < R, then:



X ∞
X ∞
X ∞
X ∞
X
n 2n 2n−1 2n
f (x) = an x = a2n x + a2n−1 x = f (−x) = a2n x − a2n−1 x2n−1 .
n=0 n=0 n=1 n=0 n=1
P∞ 2n−1
P∞ 2n−1
It follows that n=1 a2n−1 x = 0. Observe that n=1 0 · x = 0. By the Uniqueness Theorem 9.4.13,
a2n−1 = 0 for all n ∈ N.


Problem 9.4-11. Prove that if f is defined for |x| < r and if there exists a constant B such that f (n) (x) ≤ B for all |x| < r,
then the Taylor series expansion

X f (n) (0) n
x
n=0
n!

converges to f (x) for |x| < r.

Solution: According to Taylor’s Theorem with the Taylor polynomial evaluated at zero, Rn (x) = [f (n+1) (c)/(n + 1)]xn+1
where c ∈ R and c is between 0 and x. As a result, |c| < |x| < r. Therefore:

Bxn+1
0 ≤ |Rn (x)| ≤ .
(n + 1)!

Page 8
Because lim(xn+1 /(n + 1)!) = 0, it follows from the Squeeze Theorem that lim Rn (x) = 0. Consequently, the Taylor
series converges to f in the limit for |x| < r:

X f (n) (0) n
f (x) = x .
n=0
n!

Rx 2
Problem 9.4-17. Find a series expansion for 0
e−t dt for x ∈ R.

Solution: From Example 9.4.14(b), we have:


∞ ∞
2 X 1 X (−1)n 2n
e−t = (−t2 )n = t .
n=0
n! n=0
n!
By Theorem 9.4.11, the integration of interest is equal to the term-by-term integration of this power series. We then
have:
x ∞
xX ∞ Z
(−1)n 2n X x (−1)n 2n
Z Z
2
e−t = t = t
0 0 n=0
n! n=0 0
n!
∞ ∞
X (−1)n 2n+1 x X (−1)n
= t = x2n+1 ,
n=0
n!(2n + 1) 0
n=0
n!(2n + 1)
thus expressing the integration as a power series expansion.

Page 9

You might also like